Rotierender Bezugsrahmen

Können Sie mir dabei helfen:

Zwei Bezugsrahmen S Und S ' einen gemeinsamen Ursprung haben Ö Und S ' rotiert mit konstanter Winkelgeschwindigkeit ω gegenüber S .
Ein quadratischer Reifen A B C D ist aus feinem glattem Draht und hat Seitenlänge 2 A . Der Reifen ist horizontal und rotiert mit konstanter Winkelgeschwindigkeit ω um eine vertikale Achse durch A . Am Mittelpunkt der Flanke ruht zunächst eine kleine Perle, die auf dem Draht gleiten kann B C . Achsen relativ zum Reifen wählen und lassen j sei der Abstand der Perle vom Scheitelpunkt B auf der Seite B C . Notieren Sie den Positionsvektor der Perle in Ihrem rotierenden Rahmen. Zeige, dass
j ¨ ω 2 j = 0 mit dem Ausdruck für die Beschleunigung. Finden Sie daher die Zeit, die die Perle braucht, um einen Scheitelpunkt zu erreichen C .

Das habe ich gezeigt D 2 R D T 2 = ( D 2 R D T 2 ) ' + 2 ω × ( D R D T ) ' + ω × ( ω × R ) Wo ' zeigt an, dass es im rotierenden Rahmen gemacht wird. R ist der Positionsvektor eines Punktes P vom Ursprung gemessen.

ich habe das verstanden
R = R cos θ ich + j J
R ' = ( R ˙ C Ö S θ R θ ˙ Sünde θ ) ich + j ˙ J
R = ( R ¨ C Ö S θ R ˙ θ ˙ S ich N θ R ˙ θ ˙ Sünde θ R θ ¨ Sünde θ R θ ˙ 2 C Ö S θ ) ich + j ¨ J
ω × R ' = ω j ˙ ich + ( ω R ˙ cos θ ω R θ ˙ Sünde θ ) J
ω × ( ω × R ) = ω 2 R cos θ ich ω 2 j J

Ich nehme an, ich muss jetzt das zweite Newtonsche Gesetz schreiben, aber ich weiß nicht, welche Kräfte ich in dieser Bewegung habe.

Deine Gleichungen sind ziemlich schwer nachzuvollziehen. Was ist θ in Gleichung eins? Und wie hast du geschrieben R = R cos θ ich + j J ? Beachten Sie, dass j ist der Abstand der Sicke aus B und nicht von der x-Achse, also denke ich, dass diese Gleichung falsch ist. Vielleicht ist deine Herangehensweise an das Problem nicht richtig. Ich kenne einen anderen Weg, dies zu lösen. Wenn Sie möchten, werde ich es posten.
θ ist ein Winkel zwischen Positionsvektor und x-Achse im rotierenden Rahmen. Ich sollte Äxte wählen X Und j so dass sie an den Seiten des Quadrats liegen. Ich habe das gewählt X liegt mit A B Und j liegt mit A C . Ja, bitte schreiben Sie Ihre Lösung.

Antworten (1)

Ich sehe jetzt das Problem mit deiner Gleichung.

Beim Differenzieren R = R cos θ ich + j J , hast du überlegt R konstant sein, was falsch ist.
R wird von gegeben

R = l 2 + j 2
Wo l ist die Seitenlänge des Quadrats. So R wird sich mit ändern j , und Sie müssen differenzieren R zu.
Hier wird die Mathematik ziemlich hässlich und abschreckend!

Um dies zu vermeiden, können wir beobachten, dass die Perle im Rotationsrahmen eine nach außen gerichtete Zentrifugalkraft erfährt. Diese Kraft wird eine Komponente haben B C . Diese Komponente kann geschrieben werden als (ich werde Ihre Variablen ausleihen)

F B C = M ω 2 R Sünde θ
F B C = M ω 2 l 2 + j 2 j l 2 + j 2

Also durch dividieren durch M auf beiden seiten bekommt man

j ¨ = ω 2 j

Beachten Sie, dass dies dasselbe ist wie eine Bewerbung D 2 R D T 2 = ( D 2 R D T 2 ) ' + 2 ω × ( D R D T ) ' + ω × ( ω × R ) . Es ist nur so, dass dieser Ansatz problemspezifischer ist (und auch viel einfacher!).

Das Problem ist, dass ich diese Formel für die Beschleunigung verwenden MUSS. Ok, ich werde es noch einmal differenzieren, wenn ich das bedenker = r ( t ) und ich werde sehen, was ich bekomme. Sagen Sie mir einfach Folgendes: Welche Kräfte wirken auf eine Perle? Gravitation und das, was du geschrieben hast? Sind sie beide senkrecht zur vertikalen Achse? Wenn ja, kann ich den Ausdruck für Newtons zweites Gesetz mit 1 multiplizierenDR⃗ Dθ zu bekommen0 auf der rechten Seite.
Das Newtonsche Gesetz würde hier nicht viel helfen. Dies liegt daran, dass die Schleife durch ein externes Mittel eingeschränkt wird , um eine konstante Winkelgeschwindigkeit aufrechtzuerhalten. Nun ändert sich diese Kraft entsprechend dem sich ändernden Trägheitsmoment des Systems, und sie ändert sich so, dass die Schleife eine konstante Winkelgeschwindigkeit beibehält. Die Bewegungsgleichungen, die Sie haben, werden ausreichen, um dieses Problem zu lösen.
Wenn Sie immer noch nicht zufrieden sind, können Sie es sich so vorstellen. Alles, was die Gleichung des zweiten Newton-Gesetzes Ihnen geben wird, ist eine Beschreibung der Perlenbewegung, die Sie bereits bei sich haben. Differenzieren Sie einfach die GleichungsaufnahmeR als Variable und Sie sollten Ihre Antwort erhalten.
Ich habe die betreffenden Ausdrücke korrigiert. Warum gibt es in Ihrer Antwort keine Gravitationskraft? Warum beobachten Sie nur die Zentrifugalkraft?
Ich habe es. Die Gravitationskraft hat keine Komponentej -Achse.
Stimmen Sie zu, wenn ich nehme
R⃗ = r cosθich⃗ + jJ⃗ 
als richtig, würde es das bedeutenr cosθ gleich wäre2 ein seitj ist der Abstand der Perle vom ScheitelpunktB ? Wenn die Perle nicht an der Seite istBC _ diese Gleichung wird nicht akzeptabel sein. In diesem Fall wäre die richtige Gleichung
R⃗ = r cosθich⃗ + r SündeθJ⃗ 
.
Also wenn ich nur den Fall beobachte, wenn der Wulst auf der Seite istBC _ , Ich kann schreiben
R⃗ = 2 einich⃗ + jJ⃗ 
R⃗ ˙=j˙J⃗ 
R⃗ ¨=j¨J⃗ 
ω⃗ ×R⃗ ˙= −ω _j˙ich⃗ 
ω⃗ × (ω⃗ ×R⃗ ) = ω yich⃗ + 2 ein ωJ⃗ 
D2R⃗ DT2= ( 2 wj˙+ 2 einω2)ich⃗ + (j¨ω2j)J⃗ 
Du kannst nur schreibenR⃗  im Drehrahmen alsR⃗ = 2 einich⃗ + jJ⃗  , Im Trägheitsrahmen müssten Sie verwendenR⃗ = r cosθich^+ r SündeθJ^
Aber wenn ich nur den Fall beobachte, wenn die Perle auf dem istBC _ Seite und wenn ich benutzeD2R⃗ DT2= (D2R⃗ DT2)'+ 2ω⃗ × (DR⃗ DT)'+ω⃗ × (ω⃗ ×R⃗ ) , darf ich schreibenR⃗ = 2 einich⃗ + jJ⃗  ?R⃗ = 2 einich⃗ + jJ⃗  ist ein Positionsvektor in einem rotierenden Rahmen.
ja du kannst schreibenR⃗  dieser Weg
In diesem Fall habe ich
D2R⃗ DT2= ( 2 ωj˙2 einω2)ich⃗ + (j¨ω2j)
Was nun damit machen? Wie kann ich zeigen, dass es keine aktive Kraft in Richtung gibtJ⃗  wenn es eine Komponente der Zentrifugalkraft gibt?
Ist Zentrifugalkraft vielleicht schon im Ausdruck für enthaltenMD2R⃗ DT2 alsMω⃗ × (ω⃗ ×R⃗ ) ?